Math, asked by takharraman88, 3 months ago

hcf of two numbers be 40 then which of following cannot be their Lcm(1) 20(2)40(3)80(4)160​

Answers

Answered by asifkhan070305
2

Step-by-step explanation:

.2).40 cannot be their LCM..

Answered by jazharoon
1

Answer:

(1) 20,(3)80,(4)160 are not the answers

it's answer is (2) 40

Similar questions